¿Una esfera giratoria cargada uniformemente generaría un campo magnético?

Podía ver cómo iría de cualquier manera.

Por un lado, HAY carga en movimiento; por otro lado, no se está moviendo perceptiblemente.

Respuestas (3)

Lo que usted llama movimiento "perceptiblemente", se llama densidad de corriente no estacionaria. Considere un alambre con un flujo de carga constante en cada sección transversal. Entonces, esta corriente no se mueve "perceptiblemente" en el mismo sentido que su carga en la esfera. Todavía hay un campo magnético alrededor del cable.

Puedes pensar en la esfera cargada como una cantidad infinita de cables circulares alrededor del eje de rotación. Todos estos cables crean campos magnéticos que se suman. Entonces sí, hay un campo magnético debido a una esfera giratoria.

Para un enfoque de cálculo, sugiero esto: https://physics.stackexchange.com/a/173022/75518

Entonces, un electrón 'giratorio' NO produciría un campo magnético, porque (como fuente puntual singular) no representa una corriente que fluye. ¿Es esto correcto?
@Jiminion: No existe una noción como "punto de inflexión". Aún así, los electrones tienen una propiedad llamada Spin , que es un poco similar al momento angular (esto es lo que quieres decir con girar), ¡pero no es lo mismo! los electrones tienen un momento magnético debido a este giro , pero generalmente no se explica sobre una base clásica. Si te metes en la ecuación de dirac , verás que este momento magnético en realidad no es causado por ningún "giro" en absoluto. Aún así, a veces esta visualización es útil.
¿El OP significaba teóricamente solo en el espacio o en algún lugar aquí en la Tierra? Estaba bajo la suposición de que la tierra genera su propio campo magnético; por su núcleo externo moviéndose en relación con su núcleo interno sólido. ¿Esta esfera no tiene que girar en relación con algo ? (y si lo es, entonces la respuesta es, ¿sí?) ¿O si está cargado, tendría un MF, incluso sin girar?
@Mazura: Tienes razón, el movimiento siempre es relativo a algo. Aquí, se cree que la esfera gira en relación con el sistema (inercial) de un observador, donde el centro de masa de la esfera no está en movimiento. Además, nociones como campos eléctricos o magnéticos también son relativas a un observador. Piense en una carga, que está en reposo en relación con un observador. Esto significa que solo hay un campo electroestático. Ahora considere un segundo observador que está en movimiento uniforme en relación con el primero. ¡Verá la carga en movimiento uniforme y así observará un campo magnético y eléctrico!

Las cosas que inducen un campo magnético son una corriente o un campo eléctrico cambiante (con respecto al tiempo). En esta situación, tiene cargas en movimiento, por lo que tiene un J.dS, lo que significa que tiene corriente.

EDITAR:

Si prefiere un enfoque diferente, considere la esfera como una cantidad infinita de circuitos circulares (cada uno con un radio diferente debido a la superficie esférica).

Entonces sabe que cuando tiene cargas en movimiento en un circuito, significa que tiene corriente y, por lo tanto, tiene un campo magnético inducido.

Así que ahora tienes infinitos circuitos circulares uno al lado del otro. (Piense en un solenoide con giros infinitos. Aquí se puede hacer el mismo enfoque con una geometría diferente, aunque solo por intuición).

Tienes una carga en movimiento y un movimiento circular que es una aceleración para lo que necesitas una fuerza. Recuerda la fuerza de Lorentz F = q v × B . Este producto vectorial vectorial se puede reescribir como

B = F × q v q v 2
.

Editar: la ecuación se editó siguiendo https://math.stackexchange.com/questions/1219103/how-to-rewrite-a-vector-cross-product/1219127#1219127

¡Estoy seguro de que esta identidad es incorrecta! tomar vectores a = ( 1 , 1 , 1 ) , b = ( 2 , 0 , 0 ) , C = ( 0 , 2 , 2 ) después a × b = C pero C × a b y C × b a . Es más: C × a y C × b ni siquiera son (anit-)paralelos a b y a
(Dado que esto ha sido marcado) Aunque MarcelKöpke tiene toda la razón en que esta identidad es incorrecta, esta respuesta es un intento de responder la pregunta. No marque las respuestas incorrectas como "no es una respuesta" o "de baja calidad", sino que las vote negativamente.
@MarcelKöpke: Vuelva a hacer sus cálculos con vectores ortogonales.
pero B y v no son ortogonales en general. B es el campo magnético, por ejemplo, inducido por una corriente, y v es la velocidad de una partícula cargada que vuela en una dirección arbitraria (por ejemplo, esta partícula no es "parte" de la corriente)
@MarcelKöpke: haga un boceto del dispositivo a partir de la pregunta anterior. ¿Es este el caso especial de los vectores ortogonales?
El punto es que Lorentz-foce es la fuerza aplicada a una partícula que "interactúa" con el dispositivo anterior por medio de su campo magnético. Por lo general, uno no lo aplicaría a las partículas del dispositivo en sí (ya que de todos modos se mantienen una cierta rotación).
@MarcelKöpke ¿Por qué no aplicar la alineación del giro de los electrones con la rotación de un cilindro y, de esta manera, los momentos dipolares magnéticos también se alinearán? Y este es el campo magnético que describe en su respuesta. Por favor prueba ambas formas de cálculo.
@HolgerFiedler su fórmula daría lo necesario B campo para soportar las cargas que se mueven en círculos, como si la esfera cargada en sí misma no tuviera integridad estructural. Lo cual no es relevante para la pregunta que se hizo. (Es por eso que la gente dice que está equivocado: "F" es irrelevante para la pregunta, por lo que no hay razón para ello " F q v " debería significar cualquier cosa, y mucho menos ser cero).
@NeuroFuzzy: Oh, ¿entonces la esfera no significa un cuerpo rígido? Pero si sigo la respuesta de Marcel Köpke, hay un enlace a physics.stackexchange.com/questions/173019/… . Esta pregunta es sobre un cuerpo rígido. . . Me alegró tu comentario, pero ahora estoy en problemas nuevamente debido al enlace.
@HolgerFiedler SÍ significa cuerpo rígido, pero todas las FUERZAS involucradas son suministradas por la integridad estructural del campo. Tu ecuación no tiene nada que ver con el campo realmente generado por las cargas. La ley de fuerza de Lorentz no te da el campo generado por las cargas.
@NeuroFuzzy: si un electrón gira alrededor de un eje, el electrón siente una fuerza centrífuga, se acelera. Al estar acelerado y en movimiento se tiene la situación de un campo magnético inducido. Trato de ver el caso especial de la fuerza de Lorentz siempre en contexto con la inducción electromagnética y sus tres fenómenos: accionamiento EM, generador EM y bobina EM. De acuerdo con la ecuación B=... se debe generar un campo magnético paralelo al eje de rotación. Esto se corresponde con la respuesta de Marcel.
@MarcelKöpke: si un electrón gira alrededor de un eje, el electrón siente una fuerza centrífuga, se acelera. Al estar acelerado y en movimiento se tiene la situación de un campo magnético inducido. Trato de ver el caso especial de la fuerza de Lorentz siempre en contexto con la inducción electromagnética y sus tres fenómenos: accionamiento EM, generador EM y bobina EM. De acuerdo con la ecuación B=... se debe generar un campo magnético paralelo al eje de rotación. Esto se corresponde con su respuesta.
@ACuriousMind: si un electrón gira alrededor de un eje, el electrón siente una fuerza centrífuga, se acelera. Al estar acelerado y en movimiento se tiene la situación de un campo magnético inducido. Trato de ver el caso especial de la fuerza de Lorentz siempre en contexto con la inducción electromagnética y sus tres fenómenos: accionamiento EM, generador EM y bobina EM. De acuerdo con la ecuación B=... se debe generar un campo magnético paralelo al eje de rotación. Esta respuesta de los corresponsales con Marcels.
@HolgerFiedler, el campo magnético de una capa esférica giratoria cargada es exactamente como un dipolo puntual fuera de la superficie: hep.princeton.edu/~mcdonald/examples/rotatingshell.pdf y no está "a lo largo del eje z", y la respuesta de Marcel no Establece eso, afirma eso. Las ecuaciones que está usando están confundidas y se usan sin sentido. La ecuacion × B = m 0 j contiene una física diferente a la ley de fuerza de Lorentz.
Hay dos tipos diferentes de ecuaciones en electrodinámica:
1) Las ecuaciones de Maxwell , que (dadas una densidad de carga y una densidad de corriente) te dicen cuál es el campo eléctrico y magnético. También te dicen cómo E y B se influyen mutuamente (por ejemplo, ondas EM).
2) Fuerza de Lorentz , que dado (!!!) un campo eléctrico y magnético te dice qué fuerzas actúan sobre una carga q con velocidad v . No es adecuado para calcular los campos.